General Chemistry 111

Réussis tes devoirs et examens dès maintenant avec Quizwiz!

First ionization energy vs Second ionization energy

Ionization energy is the energy needed to remove the frist electron, the first IE will always be smaller than that of the second IE and will always be smaller than the third IE If losing a certain number of electrons gives an element a noble gas-like electron configurations then removing a subsequent electron will cost much more energy.

Intermolecular forces

London forces- Weakest Dipole-Dipole interactions- Intermediate strength Hydrogen bond- Strongest Ionic bond -Stronger than any Largest forces mean highest boiling points

Atomic mass vs. Atomic weight

Mass= slightly less than the sum of Protons and Neutrons of the element. Weight= Weighted avg of the isotopes of given element

mnemonic device to help with what loses and gains electrons

MeTals lose electrons to become- CaTions Nonmetals gain electrons to become aNions-

Memorize this dilution equation

MiVi = MfVf -> this can be manipulated to find any of the missing amount Vi = MfVf/Mi for exampl (1.2M)(300mL)/(5.5M) = 65mL

You are working in a sewage treatment facility and are assaying chlorine in a water sample. You need to dilute the water sample from 100ppm stock to 25ppm and create 100 mL of solution. Calculate the amount of stock solution needed and determine how you would create your final solution:

MiVi=MfVf -> MfVf/Mi = (25ppm)(100mL)/(100ppm) = 25mL Thus, start with 25mL of the stock solution and add 75 mL pure water to get 100 mL of solution with 25 ppm Cl2

Utilizing Arrhenius acid naming trends predict the acid formula and name for the following anions: MnO4- Titanate (TiO3^2- I^- IO4^-

MnO4- as acid = HMnO4, Permanganic acid Titanate (TiO3^2-) as acid = H2TiO3, Titanic acid I^- as acid = HI, Hydroiodic acid IO4^- as acid = HIO4, Periodic acid

How are molality and molarity related for water? How are they related for other solvents? Water: Other solvents:

Molarity (M) and molality (m) are nearly equal at room temperature. This is only because 1 L solution is approximately equal to 1 kg solvent for dilute solutions . For other solvents, molarity and molality differ significantly because their densities are not 1 g/mL like water.

Molarity

Molarity (moles of solute per liters of solution) is the most common unit for concentration and is used for rate laws, the law of mass action, osmotic pressure, pH and pOH, and the Nernst equation. M = moles of solute / liters of solution

Mole fraction

Mole fraction (moles of solute per total moles) is used for calculating vapor pressure depression and partial pressures of gasses in a system. XA= moles of A / total moles of all species

Most dissolutions are what?

Most dissolutions are endothermic, although the dissolution of gas into liquid is exothermic.

What is observed in a polyvalent acid or base titration?

Multiple buffering regions and equivalence points are observed in polyvalent acid and base titrations

Try drawing the resonance structures for NCO-

N could posses 6 electrons then one bond to C and a bond going from C to O N will have 4 electrons then two bonds to C and two bonds from C to O

Why are BF3 and NH3 geometries are different?

NH3 have four groups, three atoms and one lone pair. this leads to assume tetrahedral electronic geometry yet trigonal molecular geometry BF3- has there atoms and no lone pairs resulting in sp2 hybridization. Shape is tetrahedral configuration, trigonal planar in shape

Net ionic equations

Net ionic equations ignore spectator ions to focus only on the species that actually participate in the reaction. To obtain a net ionic reaction, substrate the ions appearing on both sides of the reaction, which are called spectator ions. -for reactions that contain no aqueous salts, the net ionic equation is generally the same as the overall balanced reaction -For double displacement (metathesis) reactions that do not form a solid salt, there is no net ionic reaction because all ions remain in solution and do not chain oxidation number,

What do neutralization reactions form?

Neutralization reactions form salts and sometimes water.

How do metals and nonmetals achieve their octet?

Nonmetals gain electrons while metals lose electrons to achieve their octet.

Normality

Normality (number of equivalents per liters of solution) is the molarity of the species of interest and is used for acid-base and oxidation-reduction reactions. m = moles of solute / kilograms of solvent

What is the normality?

Normality is the concentration of acid or base equivalents in solution.

Calculate the normality the following solutions: 2M Al(OH)3 16M H2SO4

Normality of 2M Al(OH)3 = 6N Normality of 16M H2SO4 = 32N

Rank the following elements by decreasing electronegativity Sb Ne O Ti

O<Sb<Ti<Ne

What is oxidation and reduction?

Oxidation (loss of electron) Reduction (gain of electrons)

OIL RIG

Oxidation Is Loss of electrons, Reductin

Oxidation & Reduction

Oxidation is a loss of electron, and reduction is a gain of electron: the two are paired together in what is known as an oxidation-reduction (redox) reaction.

Chapter 11

Oxidation-Reduction Reactions

Oxidation-Reduction titrations

Oxidation-Reduction titrations are similar in methodology to acid-base titrations. These titrations follow transfer of charge. -Indicators used in such titrations change color when certain voltages of solution are achieved

Identify the oxidation and reduction half reactions in the following redox reaction Zn + Cu2+ -> Zn2+ + Cu

Oxidation; Zn -> Zn^2+ + 2 e- Reduction; CU^2+ + 2 e- -> Cu

There are many ways of expressing concentration?

Percent composition by mass Mole fraction Molarity Molality Normality

Percent composition by mass

Percent composition by mass (mass of solute per mass of solution times 100%) is used for aqueous solutions and solid-in-solid solutions. Mass of solute / Mass of solution x 100

For a reaction involving a strong base and a weak base, which of the following indicators would be best to indicate the endpoint of the titration? Phenolpthalein (pKa = 9.7) Bromothymol blue (pKa= 7.1) Bromoscresol green (pKa= 4.7) Methyl yellow (pKa = 3.3)

Phenolphthalein is the one to go with.

What is the approximate value of pKa1? 1.9 2.9 3.8 4.1

A. 1.9 The first pKa in the curve can be estimated by eye. It si located hallway between the starting point (when no base had yet been added) and the first equivalence point ( the fist steep portion of the graph, around 15 mL). This point is at approximately 1.9. Notice that this region experiences very little change in pH, which is the defining characteristic of a buffer region.

11.) Which of the following will cause the greatest increase in the boiling point of water when it is dissolved in 1.00 kg H20? .46 mol calcium sulfate .54 mol iron(III) nitrate 1.09 mol acetic acid 1.11 mol sucrose

B. .54 mol iron(III) nitrate

What is the gram equivalent weight of phosphoric acid? 24.5 g 32.7 g 49 g 98 g

B. 32.7 g Gram equivalent weight is the weight in grams that releases one acid or base equivalent from a compound. Because H3PO4 contains 3 protons, we find the gram equivalent weight by dividing the mass of one mole of the species by 3. The mar mad of phosphoric acid is 98 g/mol, so the gram equivalent weight is 32.7 g.

What is the approximate value of pKa2? 3.6 4.1 5.5 7.2

B. 4.1 The value of the second pKa is found at the midpoint between the first and second equivalence points. In this curve, that corresponds to pH= 4.1. Just like the first pKa, it is in the center of a flat buffering region.

How many electrons are involved in the following half-reactions after it is balanced? Cr2O7^2- + H^+ + e- -> Cr^2+ + H2O 2 8 12 16

B. 8 #2 on page 392

Which of the following actions does NOT affect the equilibrium position of a reaction? Adding or removing heat Adding or removing a catalyst Increasing or decreasing concentrations of reactants Increasing or decreasing volume of reactants

B. Adding or removing a catalyst The equilibrium of a reaction can be changed by several factors. Adding or subtraction heat would shift the equilibrium based on the enthalpy change of the reaction. Increasing reactant concentrations would shift the equilibrium in the direction of the product, and the opposite would occur if reactant concentrations were decreased. Changing the volume of a reactant would affect any reaction with gaseous reactants or products, eliminating D. while adding or removing a catalyst would change the rate, it would not change there the equilibrium lies.

Which of the following is true of equilibrium reactions? I. An increase in K1 results in a decrease in k-1 II. As the concentration of products increases, the concentrations of reactants decreases III. The equilibrium constant is altered by changes in temperature A. I only B. II and III only C. I and III only D. I, II, and III

B. As the concentration of products increases, the concentrations of reactants decreases, The equilibrium constant is altered by changes in temperature these are true regarding equilibrium reactions Statement I is false because the addition of a catalyst could increase the rate constants of both the forward and reverse reactions. Statement II is true because - for products to come into existence - reactants must be used up. Statement III is also true; all K values are temp-dependent

Buffering capacity refers to what?

Buffering capacity refers to the ability of a buffer to resist changes in pH; maximal buffering capacity is seen within 1 pH point of the pKa of the acid in the buffer solution.

Describe each of the following parts of a titration curve; Buffering region Half-equivalence point Equivalence point Endpoint

Buffering region - occurs when [HA] = [A-] and is the flattest portion of the titration curve (resistant to changes in pH) Half-equivalence point - is the center of the buffering region Equivalence point - is the steepest point of the titration curve and occurs when the equivalents of acid present equal the equivalents of base added (or vice - versa) Endpoint - is the pH at which an indicator turns its final color

What is the approximate pH of a 1.2 x 10^-5 M aqueous solution of NaOH? 4.92 7.50 9.08 12.45

C 9.08 NaOH is a strong base; as such, there will be 1.2x10^-5M OH^- in solution. Based on this information alone, the pOH must be between 4 & 5, and the pH must be between 9 & 10. Using the shortcut, pOH = 5 - .12 = 4.88. pH = 14 - pOh = 9.12

Wha is the oxidation number of chlorine in NaClO? -1 0 +1 +2

C. +1 #4 on 392

Consider the following two reactions: 3A + 2B = 3C + 4D (reaction 1) 4D + 3C = 3A + 2B (reaction 2) If Keq for reaction 1 is equal to .1 what is Keq for reaction 2? .1 1 10 100

C. 10 Reaction 2 is simply the reverse of reaction 1. this means the Keq for reaction 2 is the inverse of Keq of reaction 1, so the answer is (0.1)^-1 = 10

How many liters of 2M Ba(OH)2 are needed to titrate a 4L soliton of 6M H3PO4? 1.33 L 12 L 18 L 56 L

C. 18 L Look on 367

Consider the following equation 6Na (s) + 2 NH3 (aq) -> 2 Na3N (s) + 3 H2 (g) Which species acts as an oxidizing agent? Na N in NH3 H in NH3 H2

C. H in NH3 The oxidizing agent is the species that is reduced in any given equation. In this problem, six hydrogen atoms with +1 oxidation sates in NH3 are reduced to three neutral H2 Molecules.

A solution is prepared with an unknown concentration of a theoretical compound with a Ka of exactly 1.0. What is the pH of this solution? Higher than 7 Exactly 7 Less than 7 There is not enough information to answer the question.

C. Less than 7 A higher Ka implies a stronger acid. Weak acids usually have a Ka that is several orders of magnitude below 1. The pKa of a compound is the pH at which there are equal concentrations of acid and conjugate base; the pKa of this compound would be -log 1= 0. With such a low pKa, this compound must be an acid. Therefore, the pH of any concentration of this compound must be below 7.

4.)Which of the following explanations best describes the mechansim by which solute particles affect the melting point of ice? Melting point is elevated because the kinetic energy of the substance increases Melting point is elevated because the kinetic energy of the substance decreases Melting point is depressed because solute particles interfere with lattice formation Melting point is depressed because solute particles enhance lattice formation.

C. Melting point is depressed because solute particles interfere with lattice formation Melting point depresses upon solute addition, making A & B incorrect. Solute particles interfere with lattice formation, the highly organized state in which solid molecules align themselves. Colder-than-normal conditions are necessary to create the solid structure.

What are the configurations of Chromium and Copper

Chromium you would think would be 4s^2, 3d^4. However moving one electron from the s shell to the d shell would give you 4s^1, 3d^5 Copper you would think would be 4s^2, 3d^9 but it is actually 4s^1, 3d^10

Which of the following elements has the highest electronegativity? Mg Cl Zn I

Cl electronegativity increases as one moves from left to the right for the same reasons that effective nuclear charges increase Cl is the furthest toward the top right corner

Draw the next couple of example and guess the Electronic geometry: BeCl2

Cl-Be-Cl linear

- Hypo = less and -per = more oxygen

ClO- = Hypochlorite ClO2- Chlorite ClO3- Chlorate ClO4- Perchlorate

Colligative properties are what?

Colligative properties are physical properties of solutions that depend on the concentration of dissolved particles but not on their chemical identity. -vapor pressure depression -boiling point elevation -freezing point depression -osmotic pressure

What is a colligative property?

Colligative properties are those that depend on the amount of solute present, but not the actual identity of the solute particles. Examples include vapor pressure depression, boiling point elevation, freezing point depression and osmotic pressure.

Comparison of Q to Keq provides what information?

Comparison of Q to Keq provides information about where the reaction is with respect to its equilibrium state. -if Q < Keq, (delta) G < 0 and the reaction proceeds in the forward direction -If Q = Keq, (delta) G = 0 and the reaction is in dynamic equilibrium -If Q > Keq, (delta) G > 0 and the reaction proceeds in the reverse direction

What is the name of the bond in Carbon Monoxide? Ionic Polar covalent Nonpolar covalent Coordinate covalent

Polar covalent C-O has a triple bond between the C and O, with the carbon and the oxygen each retaining one lone pair. -Oxygen is more electronegative than Carbon so the electrons will be disproportionately carried on the oxygen, leaving the carbon atom with a slight positive charge

Polyvalent acids and bases are what?

Polyvalent acids or bases are those that can donate or accept multiple electrons. The normality of a solution containing a polyvalent species is the molarity of the acid or base times the number of protons it can donate or accept.

Factors affecting reaction rates

Reaction Concentrations Temperatures Medium Catalysts

Reactions may have both kinetic and thermodynamic products that can be regulated by what?

Reactions may have both kinetic and thermodynamic products that can be regulated by temperature and the presence of a catalyst.

In which part of the pH range (acidic, basic or neutral) will the equivalence points fall for each of the following titrations? Strong acid + Weak base Strong base + Weak acid Strong acid + Strong base Weak acid + Weak base

Strong acid + Weak base- have an equivalence point in the acidic range. Strong base + Weak acid - have an equivalence point in the basic range Strong acid + Strong base - have an equivalence point at pH = 7 (neutral) Weak acid + Weak base- can have an equivalence point in the acidic, neutral, or basic range, depending on the relative strengths of the acid or base.

Key concept

Strong acid + weak bases ; equivalence point pH < 7 Strong acid + Strong base ; equivalence point pH = 7 Weak acid + Strong base ; equivalence point pH >7

Strong Acids and bases do what?

Strong acids and bases dissociate completely in solution.

Strong or weak electrolytes

Strong electrolyte if it dissociates completely into its constituent ions. Examples such a NaCl and KI or HCl in water. Weak electrolyte on the other hand ionizes or hydrolyzes incompletely in aqueous solution and only some of the solute it dissolved into its ionic constituents. Examples: Hg2I2, Acetic acid, weak acid, Ammonia and other weak bases Non-electrolytes: O2, CO2 and glucose

Describe the effects of the following condone would have on the initial rate of reaction, given the reaction order Temp lowered --> 0-order, 1st-order, 2nd order All reactants concentrations double -- 0-order, 1st order, 2nd order Catalyst added --> 0-order, 1st order, 2nd order

Temperature lowered decreases the rate on 0-order ,1st-order, 2nd-order If reactants concentrations double it has zero effect on zero-order, if the reactants concentrations double it double the rate of 1st-order If the reactants concentrations double the rate is multiplied by 4 in 2nd-order If catalyst is added it increases the rate of all Zero-order, 1st-order, and 2nd-order

CH4

Tetrahedral

What is the activation energy?

The activation energy is the minimum energy needed for a chemical reaction to occur.

Describe in words what is occuring in the following two-step mechanism. Step 1: A2B + A2B --> A4B2 Step 2: A4B2 --> 2A2 + B2

The rate is first order with respect to the concentration of the first reactant; when the concentration of the reactant doubles, the rate also doubles. Because the reaction is third order, the sum of the exponents in the rate law muse be equal to 3. Therefore the reaction order with respect to the other reactant must be 3-1=2

What does it mean for a step in a mechanism to be the rate determining step?

The rate-determining step is the slowest step of a reaction. It determines the overall rate of the reaction because the reaction can only proceed as fast as the rate at which this step occurs.

Consider the formation of HCl H2 (g) + Cl2 (g) = 2 HCl (g)

The reaction diagrams of HCl forming is exergonic, the free energy of the products is less than the free energy of the reactants; energy is released and the free energy change of the reaction is negative

Intermediate

a molecule that reacts in a reaction but does not show up in the reaction A2 + B = A2B A^2B+B = 2AB A^2B would be the intermediate

Zero-Order reactions aA + bB = cC + dD

a zero order reaction is one in which the rate of formation of product C is independent of changes in concentrations of any reactants, A & B. These have a constant reaction rate equal to the rate constant k. rate law for a zero order reaction is rate= k[A]^0[B]^0=k k has units of M/s K= -slope as well

pH and pOH can be calculated how?

pH and pOH can be calculated given the concentrations of H3O+ and OH- ion, respectively. In aqueous solutions, pH + pOH=14 at 298K

Traps to look for on MCAT regarding rate laws

cases in which the coefficients match the orders of the reaction- when the reaction mechanism is a single step and the balanced overall reaction is reflective of the entire chemical process or when the complete reaction mechanism is given and the rate-determining step is indicated dont mistake the equilibrium constant expression for the rate law, remember that the equilibrium expression includes the concentrations of all the species in the reaction, both reactants and products. Rate law on the other hand includes only the reactants Rate constant, k, for a reversible reaction the Keq is equal to the ratio of the rate constant for the forward reaction. last is the notion and principles of equilibrium that apply to the system only at the end of the reaction; that is after the system has already reached equilibrium.

Ionic compounds form from

combination of elements with large electronegativity difference such as sodium and chlorine

skip 6,7

ddd

angular quantum number

denoted by the letter "l" refers to the shape and number of subshells within a given principal energy level, these are important for its implications for chemical bonding and bond angles. "n" limits the value of "l" in the following way: for any given value of "n" the range of possible value for l is 0 to (n-1)

remember the octet rule?

eight electrons in the outermost shell, Argon gas example: has a complete octet in the valence shell

Molecular compounds form from

elements of similar electronegativity such as carbon with oxygen

Transition Metals

elements that form a bridge between elements on the left and right sides of the periodic table

Limiting Reagent

limits the amount of product that can be formed n the reaction Note on MCAT: If two reactants are given expect to be asked what is the Limiting reagent

Balance the following redox reaction using the half reaction method Mg (s) + HNO3 (aq) -> Mg2+ (aq) + NO (g)

look at it on 388 in chem

A/Z X determination of what is in said element

mass number (P+N) = A Atomic number (P) = Z atom = X

Mole equation

mass of sample (g) = ------------------------ Molar mass (g/mol)

Qauntum numbers

n, l , ml, and ms Values of n limited that of l and then in turn limit ml.

dont do 15

ndn

Single Displacement Reactions

occurs when an atom or ion a compound is replaced by an atom or ion of another element Cu (s) + AgNo3 (aq) --> Ag (s) + CuNO3 (aq)

Decomposition Reactions

opposite of combination a single reactant breaks down into two products 2 HgO (S) --> 2 Hg (l) + O2 (g) A --> B+C

Heterogeneous Catalysis

the catalyst is in a distinct phase

Homogeneous Catalysis

the catalyst is in the same phase as the reactants

What is the best explanation for why CO3^2- has trigonal planar electronic geometry while ClF3 has trigonal bipyramidal electronic geometry?

the central Carbon in carbonate has no lone pairs, it has three resonance structures involving a double bond from the Carbon to one of the three Oxygen in each resonance. Making Four bonds to the Carbon leaving no lone pairs. Trigonal Planar shown in picture ClF3 has three bonds however chloride still maintains two extra lone pairs. Chloride must have five times about itself three bond to fluoride and two lone pairs. Trigonal By pyramidal ClF3 has lone pairs on its central atom, while CO3^2- has none. CO3^2- can have 4 bonds on the carbon through resonance.

Electron Affinity

the energy change associated with the addition of an electron to a gaseous atom to form an anion. generally exothermic on the right side. trend similar to IE

Medium

the rate at which a reaction rakes place may also be affected by the medium in which it takes place. most reaction can take place in aqueous environments, while others are more likely to react in non aqueous solvent such as DMSO or ethanol The physical state of the medium can also have a significant effect . Polar solvents are preferred because their molecular dipole tends to polarize the bonds of the reactants thereby lengthening and weakening them, permitting the reaction to occur faster

What does comparison of the ion product (IP) to Ksp determine?

Comparison of the ion product (IP) to Ksp determines the level of saturation and behavior of the solution. IP < Ksp : the solution is unsaturated, and if more solute is added, it will continue to dissolve. IP = Ksp : the solution is saturated (at equilibrium), and there will be no change in concentration IP>Ksp; the solution is supersaturated and a precipitate will form.

Complete ionic equation

Complete ionic equation accounts for all of the ions present in a reaction. To write a complete ionic reaction split all aqueous compounds into their relevant ions. Keep solid salts intact.

Complex ions or coordination compounds are composed of what?

Complex ions or coordination compounds are composed of metallic ions bonded to various neutral compounds and anions, referred to as ligands.

What is the name of the new bond formed when going from H2O to H3O

Coordinate Covalent Bond The addition of a new bond from H+ and H2O and this is created veal the sharing of oxygens lone pairs with the free H+ ion. Hence sharing of electrons from Lewis Base (H2O) and Lewis Acid (H+) .

What is the equilibrium expression for the reaction? Cu2SO4 (s) = 2 Cu+ (aq) + SO4^2- (aq) [Cu+]^2 [SO4^2-]/ [Cu2SO4] 2x[Cu+][SO4^2-]/[Cu2SO4] [Cu+][SO4^2-] [Cu+]^2[SO4^2-]

D, [Cu+]^2[SO4^2-] Recall that pure solids and liquids do not appear in the equilibrium expression; thus, this Keq has no denominator because the only reactant is a solid, cuprous sulfate. This could also be called Kip because a solid is dissociating into ions in solution. The correct Keq should have [Cu+] squared because its stoichiometric coefficient is 2.

Which of the following is not a bronsted-lowry base? Look at 1 on 364

D. A bronsted-Lowry base is defined as a proton acceptor, Ammonia, Fluoride, and water each accept a proton. HNO2 is a far better Bronsted-Lowry acid, donating a proton to solution.

What is the pH of a solution with an ammonium concentration of 70mM and an ammonia concentration of 712 mM? 2.45 4.45 9.55 11.55

D. 11.55 Look at explanation on 366

After balancing the following oxidation-reduction reaction, what is the sum of the stoichiometric coefficients of all of the reactants and products? S8 (s) + NO3^- (aq) -> SO3^2- (aq) + NO (g) 4 50 91 115

D. 115 14 393

The following electron configurations represent elements in their neutral form. Which element is the strongest oxidizing agent? 1s^2,2s^2, 2p^6.3s^2,3p^6, 4s^2 1s^2,2s^2, 2p^6.3s^2,3p^6, 4s^2,3d^5 1s^2,2s^2, 2p^6.3s^2,3p^6, 4s^2,3d^10,4p^1 1s^2,2s^2, 2p^6.3s^2,3p^6, 4s^2 ,3d^10,4p^5

D. 1s^2,2s^2, 2p^6.3s^2,3p^6, 4s^2 ,3d^10,4p^5 #5 392

What is the [H3O] of a 2M aqueous solution of a weak acid HXO2 with Ka= 3.2 x 10^-5 6.4 x 10^-5 M 1.3 x 10^-4 M 4.0 x 10^-3 M 8.0 x 10^-3 M

D. 8.0 x 10^-3 M look on 367 # 11

Which of the following is NOT a characteristic of an amphoteric species? Amphoteric species can act as an acid or a base, depending on its environment Amphoteric species can act as an oxidizing or reducing agent, depending on its environment Amphoteric species are sometimes amphiprotic Amphoteric species are alway non polar

D. Amphoteric species are alway non polar An Amphoteric species is one that can act either as an acid or a base, depending on its environment. Proton transfers are classic oxidation-reduction reactions, so A and B are true. C is true because many amphoteric species, such as water and bicarbonate can either donate or accept a proton. D is false and thus the correct answer becasue amphoteric species can be either polar or non polar in nature.

The Kip of Ba(OH)2 is 5x10^-3. Assuming the barium hydroxide is the only salt added to form a solution, calculate the ion product of the following solutions based on the concentrations of Ba2+. Then, predict the behavior of the given solutions (dissolution, equilibrium, or precipitation. Ba2+ .5 M .1 M .05M

Ion product .5 M = (.5M)(1M)^2 = .5 .1M = (0.1M)(0.2M)^2 = 4.0 x 10^-3 .05M = (0.05 M)(0.1 M)^2 = 5.0 x 10^-4 Behavior of solution .5M = .5 > 5 x 10^-3 -> precipitation .1M = 4 x 10^-3 < 5 x 10^-3 --> dissolution 0.05M = 5 x 10^-4 < 5 x 10^-3 --> dissolution

Changing pressure on a gaseous system does what?

Increasing pressure on a gaseous system (decreasing its volume) will shift the reaction toward the side with fewer moles of gas. Decreasing pressure on a gaseous system (increasing its volume) will shift the reaction toward the side with more moles of gas.

Changing the concentration does what?

Increasing the concentration of reactants or decreasing the concentration of products will shift the reaction to the right. Increasing the concentration of products or decreasing the concentration of reactants will shift the reaction to the left.

What would happen in this situation: H2SO4 (aq) = H^+ (aq) + HSO4- (aq) the pH has been increased

Increasing the pH of H2SO4 (aq) = H^+ (aq) + HSO4- (aq): [H+] decreases, shifting reaction to the right

Changing the temperature of an endo/exo-thermic reaction

Increasing the temperature of an endothermic reaction or decreasing the temperature of an exothermic reaction will shift the reaction to the right. Decreasing the temperature of an endothermic reaction or increasing the temperature of an exothermic reaction will shift the reaction to the left.

What are indicators?

Indicators are weak acids or bases that display different colors in their protonated and deprotonated forms -this indicator chosen for a titration should have pKa close to the pH of the expected equivalence point -the endpoint of a titration is when the indicator reaches its final color

Which of the following contributes the most to NO2's resonance hybrid? I. O-N=O = (-1, +1, 0) II. O=N-O = (0, +1, -1) III. O-N-O = (-1, +2, -1)

I. & II. These are the most stable.

Which will fill first, 5d or 6s subshell?

6S will fill first Remember if the element falls after 6 or 7 S then the 4&5 F elements will follow then 5&6 D then 4-7 F. and only if the element falls after 6 or 7 S

Look at osmium, what is its configuration?

6s^2. 4f^14, 5 d ^6

Example 400 g AlCl3 is dissolved in 1.5 L of water at room temp (Kb= 0.512 Kxkg/mol) How much doe the boiling point increase after adding the aluminum chloride?

(delta) Tb= iKbm Water at room temp as a density of 1 g/mL. Therefore, 1.5L is the same as 1.5 kg. The vant hoff factor for AlCl3 is 4 because it breaks down to from 1Al and 3 Cl. To determine molality we will also need to know how many mole 400 g AlCl3 represents. 400 g AlCl3 x 1 mol / 133.5 g = 3 mol AlCl3 molality is therefore 3 mol AlCl3/ 1.5 kg = 2m (delta) Tb= iKbm = (4) (.512Kxkg/mol) (2 M) = 4 K (actual 4.1K)

Determine the new boiling point of a solution containing 190 g MgCl2 in 1800 g water at room temperature: (Kb=.512 kxkg/mol)

(delta) Tb= iKbm = (3)(.512 kxkg/mol)[2mol MgCl2/1.8 kgwater] = 2K =actual = 1.7 but rounded up New boiling point will be 373 + 2 = 375 K

Freezing point depression

(delta) Tf= iKfm (delta) Tf = is the increase in boiling point i= vant hoff factor = number of particles into which a compound dissociates in solution. NaCl = ( i=2 ) Covalent molecules such as glucose have i values of 1 Kf= proportionality constat characteristic of a particular solvent (is provided) m = is the molality of the solution

Determine the freezing point depression of a solution containing 58.5 of NaCl of 2160 g of water at room temperature (Kf= 1.86 kxkg/mol)

(delta) Tf= iKfm =(2)(1.86 kxkg/mol)(1mol NaCl/2.16 kg water) = 1.86 K (actual= 1.72 K)

Example 400g AlCl3 is dissovled in 1.5L of water at room temperature (Kf = 1.86 K x kg/ mol) What is the new freezing point of this solution?

(delta) Tf= iKfm = (4) (1.86 K x kg/ mol)(2M) = 15K (actual 14.9 K) The normal freezing point of water is 273 K. The freezing point is going to be depressed (or decreased) by 15K. The new freezing point is therefore 273-15=258K = -15 C

Osmotic pressure

(pi symbol) = iMRT (pi) = osmotic pressure i = the van't hoff factor M= M is the molarity of the solution R= R is the ideal gas constant T= T is the temperature

Remember for problems with Limiting Reagents..

* All comparisons of reactants must be done in units of Moles. Gram to Gram comparisons will be useless and may be misleading * Rate at which the reactants are consumed, combined with the absolute mole quantities determines which reactant is the limiting reagent

A stock solution for making typical IV saline bags contain 90 g of NaCl per 10 liters of water (density= 1 g/mL). What is the mole fraction and the percent composition by mass of NaCl in the saline solutions? Mole fraction Percent composition by mass

**** this

Given that [product] = 0.075 M and [reactant] = 1.5 M, determine the direction of reaction and the sign of the free energy change for reactions with the following Keq values; 5.0 x 10^-2 5.0 x 10^-3 5.0 x 10^-1

5.0 x 10^-2: at equlibrium; no net reaction = (delta) G=0 5.0 x 10^-3: Qc > Keq: proceeds toward reactants (left) = (delta) G = postive 5.0 x 10^-1: Qc < Keq; proceeds toward products (right) = (Delta) G = negative

How many total electrons are in a 133 Cs cation?

54 55 protons and 55 electrons P and E together while the N stays out 133 - 55 = 78 neutron this is a cation meaning it lost an electron deeming it to have 54

Dipole-Dipole interactions

- Polar molecules orient themselves such that the positive region of one molecule is close to the negative region of another molecule - present in solid and liquid phases, not significant in gas because the increases distance between the particles

Covalent bond characteristics

- bond length, which decreases in length as it goes up -bond energy, which is the energy required to break a bond, the more electrons shared the more energy required to break it. Meaning triple bonds have the highest bond strength and single bonds having lowest. C-C, longest but weakest C=C, medium length and strength C(triple)C, Shortest and strongest -polarity, difference in electronegativities. The atom with the higher electronegativity gets the larger share of the electron density. a polar bond creates a dipole, with the positive end of the dipole at the less electronegative atom. and the negative end at the more electronegative atom

Alkali Metals (group 1) basics

- classic properties of metals - densities are lower than those of other metals - ONE loosely bound electron in outermost shell - Low ionization energies - Low electron affinities - Low electronegativites - react readily with nonmetals and easily loose one electron, especially with halogens oxidation state +1

Assign oxidation numbers to the atoms in the following reaction to determine the oxidizing and reducing agents SnCl2 + PbCl4 -> SnCl4 + RbCl2

-All are neutral, so all must add to zero SnCl2 = Sn = +2 because there are two Cl and has an oxidation number of -1 and on the right Sn= 4 and Cl= -4 PbCl4 = Pb= +4 PbCl2 = Pb= +2

Example question: Determine the number of protons, neutrons, and electrons in a nickel-58 and in a nickel-60 +2 cation.

58-Ni has an atomic number of 28 which are the protons and a mass number of 58. Since the mass number is 58 this is the protons and neutrons. 58- 28 equals 30 neutrons. Protons equal electrons in a normal atom so 28 protons then there is 28 electrons. 60-Ni +2 will have the same number of protons as the normal atom of Ni so it has 28 protons and it lost two electrons due to the + charge. so it will have 28 regular protons because it is still nickel, 26 electrons since it lost 2 giving it a positive charge but it had to gain 2 neutrons by the loss of electrons, resulting in 32 neutrons. 28 regular protons 26 electrons due to loss 32 neutrons due to gain by loss of electrons to keep peace in atm

How to assign oxidation numbers

-The oxidation number of a free element is zero, N2, P4, S8 and He all have oxidation numbers of 0 - The oxidation number for a monatomic ion is equal to the charge of the ion. For example the oxidation numbers for Na+, Cu^2+, Fe^3+ Cl^= and N^3- are 1+, 2+, 3+, 1- and 3- -The oxidation number of each Group IA element in a compound is +1 -The oxidation number of each Group IIA elements in a compound is 2+ -The oxidation number of each group VIIA element in a compound is -1, except when combined with an element of higher electronegativity. For example in HCl, the oxidation number of Cl is -1; in HOCl, however the oxidation number of Cl is +1 -Oxygen has oxidation number of -2, exception being peroxide O2^2- which is -1, and more electronegative elements, such as OF2 in which oxygen has a +2 charge. -The oxidation number of hydrogen is usually +1 Hydrogen is +1 in HCl but -1 in NaH

Hydrogen Bonds

-The partial positive charge of the hydrogen atom interact with the partial negative charge located on the electronegative atoms (F, O, N) of nearby molecules - Pick up the FON (phone

Rules on assigning oxidation numbers

-any free element or diatomic species has an oxidation number of zero -the oxidation number of a monatomic ion is equal to the charge of the ion -When in compounds, Group IA metals have an oxidation number of +1; group IIA have an oxidation number of +2 - When in compounds Group VIIA elements have an oxidation number of -1 (unless combined with an element with higher electronegativity) -The oxidation state of hydrogen is +1 unless it is parked with a less electronegative element, in which case it is -1 -The oxidation state of oxygen is usually -2, except in peroxides or in compounds with more electronegative elements -The sum of the oxidation numbers of all the atoms present in a compound is equal to the overall charge of the compound

What are the three main types of stresses applies to a system?

-changes in concentration -changes in pressure and volume -changes in temp.

- ite and -ate

-ite = less oxygen NO2- = Nitrite SO3^2- = Sulfite -ate = more oxygen NO3- = Nitrate SO4^2 = Sulfate

Molecular weight of SOCI2

1 S: 1 x 32.1 amu = 32.1 1 O: 1 x 16 amu = 16 2 Cl: 2x 35.5 amu = 71 Total = 119.1 amu per molecule

Example of Mole

1 molecule of H2CO3 has a mass of 62 amu and one mole has a has of 62 grams

How many moles are in 9.53 grams of MgCl2?

1 x 24.3 (g/mol) + 2x 35.5 (g/mol) = 95.3 (g/mol) 9.53/ 95.3 (g/mol) = .10 mol MgCl2

Determine the vapor pressure of a solution containing 190 g MgCl2 in 720 g water at room temperature (vapor pressure of pure water at 25 degrees celsius if 3.2pKa)

190 g MgCl2 x 1mol/95.3 g = 2 mol MgCl2. 720 g H2O x 1mol/18g = 40 mol H2O. X water = 40 moles water/ 42 total moles = .95 Pwater = XwaterPwater = (.95)(3.2) = 3.05 kPa

For each for her reactions, identify the oxidation states of the relevant atoms, the oxidizing agents, and the reducing agent. 2KI + H -> 2K + 2HI Al + BPO4 -> B + AlPO4

2 KI + H -> 2K + 2HI Oxidation numbers = (+1,-1) for KI, (0) for H, (0) for K (+1,-1) for HI Al + BPO4 -> B + AlPO4 Oxidation numbers = (0) for Al, (3) for B, (3) for PO4, (0) for B, (+3) for Al, (-3) for PO4

If the rate law for a reaction is rate= k[A]^0[B]^2[C]^1 what is the overall order for the reaction 0 2 3 4

3 The overall order of a reaction is the sum of the individual orders in the reaction. 0+2+1= 3

Stoichiometry basics

3 fractions * convert from given units to moles * Use the mole ratio * Convert from moles to the desired units

Key concept The reaction; A (aq) + 2 B (g) = C (g) + heat

A (aq) + 2 B (g) = C (g) + heat will shift right if: -A or B is added -C is removed - the pressure is increased or the volume is reduced -the temperature is reduced will shift left if: -C is added -A or B is removed -the pressure is reduced or the volume is increased -the temperature is increased

A Lewis acid is a what?

A Lewis acid is an acceptor of electron paris.

What is the purpose of a buffer solution?

A buffer solution is designed to resist change in pH and has optimal buffering capacity within 1 pH point from its pKa.

In a sealed 1 L container, 1 mole of nitrogen gas reacts with 3 moles of hydrogen gas to from 0.05 miles of NH3, at equilibrium. Which of the following is closest to the Kc of the reaction? .0001 .001 .01 1

A. .0001

At sea level and 25 degrees celsius the solubility of oxygen gas in water is 1.25 x 10^-3 M. In Denver, a city in the US that lies high above sea level, the atmospheric pressure is .800 atm. What is the solubility of oxygen in water in Denver? 1.00 x 10^-3 M 1.05 x 10^-3 M 1.50 x 10^- 3 M 2.56 x 10^-3 M

A. 1.00 x 10^-3 M The solubility of gases in liquids is directly proportional to the atmospheric pressure. This is called Henrys law. Even without the formula we know there is a direct proportionality between partial pressure and solubility. Because .800 atm is 80 % of the pressure at sea level (1 atm), oxygens solubility will be 80% of 1.25 x 10 ^-3M which is 1.00 x 10^-3

FeI (aq) + I2 (g) = FeI3 (aq) Which of the following would increase the formation of product? Decreasing the volume of the container Decreasing the pressure of the container Increasing the volume of the container Decreasing the volume of the container while maintaining a constant pressure

A. Decreasing the volume of the container Both increasing the pressure of the container and decreasing the volume would favor the side with fewer moles of gas, which is the product side. This make B & C incorrect D would not disturb the equilibrium - the significance of decreasing the volume of the container in most equilibrium is that there is an increase in pressure: in this case, however the pressure remain constant despite the change in volume

9.) Which of the following combinations of liquids would be expected to have a vapor pressure higher than the vapor pressure that would be predicted by Raoult's law? Ethanol and hexane Acetone and water Isopropanol and methanol Nitric acid and water

A. Ethanol and hexane Mixtures have a higher vapor pressure than predicted by Raoults law have stronger solvent-solvent interactions. Therefore, particles do not want to stay in solution and more readily evaporate, creating a higher vapor pressure than an ideal solution. Two liquids that have different properties, like hexane (hydrophobic) and ethanol (hydrophilic, small) in A. would not have many interaction with each other and owl cause positive deviation. B & C are composed of liquids that are similar to one another and would not show significant deviation from Raoults Law. D contains two liquids that would interact very well with each other, which would actually cause a negative deviation from Raoults law - when attracted to one other, solutes and solvents prefer to stay in liquid form and have a lower vapor pressure than predicted by Raoults law.

Which of the following have the largest dipole moment? HCN H2O CCl4 SO2

A. HCN Strong dipole moment in the direction towards Nitrogen. H2O has two dipole moments, one from each hydrogen pointing in the direction of oxygen, The structure is bent and the dipole moments partially cancel out. The molecular dipole is weaker than HCN SO2 has a bent configuration, and its dipole will again be smaller than that of HCN, Sulfur and Oxygen do not have a large difference in electronegativities CCl4 has tetrahedral geometry, this is highly polar but the orientation of these bonds cause the dipoles to cancel each other out

Which of the following represents chloric acid? HClO3- ClO3- HClO2 HClO

A. HClO3- Answering the question is simply a matter of knowing nomenclature. Acids ending in -ic are derivatives of anions ending in -ate, while acids ending in -ous are derivatives of anions ending in -ite. ClO3^- is chlorate because it has more oxygen than the other commonly occurring ion. ClO2^- which is named chlorite. Therefore, HClO3 is chloric acid. HClO2 represents chlorous acid.

Lithium aluminum hydride (LiAlH4) is often used in laboratories because of its tendency to donate a hydride ion. Which of the following roles would lithium aluminum hydride likely play in a reaction? Strong reducing agent only Strong oxidizing agent only Both a strong reducing agent and strong oxidizing agent. Neither a story reducing agent nor a strong oxidizing agent

A. Strong reducing agent only #3 on 392

Carbonated beverages are produced by dissolving carbon dioxide in water to produce carbonic acid: CO2(g) + H2O (l) = H2CO3 (aq) When a bottle containing carbonated water is opened, the taste of the beverage gradually changed as the carbonation is lost. Which of the following statements best explains this phenomenon? The change is pressure and volume cases the reaction to shift to the left, thereby decreasing the amount of aqueous carbonic acid. The change in pressure and volume causes the reaction to shift to the right, thereby decreasing the amount of gaseous carbon dioxide Carbonic acid reacts with environmental oxygen and nitrogen Carbon dioxide reacts with environmental oxygen and nitrogen.

A. The change is pressure and volume cases the reaction to shift to the left, thereby decreasing the amount of aqueous carbonic acid. Carbon dioxide gas evolves and leaves the bottle which decreases the total pressure of the reactants. Le Chateliers principle explains that a decrease in pressure shifts the equilibrium to increase the number of moles of gas present. This particular reaction will shift to the left, which in turn will decrease the amount of carbonic acid an increase the amount of carbon dioxide and water. Oxygen and nitrogen arrant highly reactive or carbonixacid as in C & D

If Kc >> 1: The equilibrium mixture will favor products over reactants The equilibrium mixture will favor reactants over products. The equilibrium concentrations of reactants and products are equal. Th reaction is essentially irreversible

A. The equilibrium mixture will favor products over reactants The larger the value of Keq (whether Kc or Kp) the larger the ratio of products to reactants. Therefore if Kc>>1, there are significantly larger concentrations of products than reactants at equilibrium. Even with a large Keq, the reaction will ultimately reach equilibrium far toward the products is and is therefore reversible, eliminating D.

Which of the following statements best describes the effect of lowering the temperature of following reactions? A + B = C +D (delta) H = -1.12 KJ/mol [C] and [D] would increase [A] and [B] would increase (delta) H would increase (delta) H would decrease

A. [C] and [D] would increase

As methanol is converted to methanal, and then methanoic acid, the oxidation number of the carbon increases decreases increases, then decreases decreases, then increases

A. increases #9 on 393

NOTE:

ALL Arrhenius acids and bases are Bronsted-Lowry acids and bases. ALL Broasted-Lowry acids and bases are Lewis acids and bases However; the converse of these statements is not necessarily true ( Not all lewis acids and bases are Bronsted-Lowry acids and bases. and not all Bronsted-Lowry acids and bases are Arrhenius acids and bases.)

Remember!

Acid-base reactions that consist of a single headed arrow generally indicate strong acids or bases

Chapter 10

Acids and bases

What species are considered the equivalents for acids and bases, respectively?

Acids use moles of H+ (H3O+) as an equvalent. Bases use moles of OH- as an equivalent

Amphiprotic species are what?

Amphiprotic species are amphoteric species that specifically behave as a Bronsted-Lowry acid or Bronsted- Lory Base.

Amphoteric species are what?

Amphoteric species are those that can behave as an acid or base.

What is an amphoteric species?

An amphoteric species can act as an acid or base

An oxidizing/reducing agent is what?

An oxidizing agent facilitates the oxidation another compound and is reduced itself in the process; a reducing agent agent facilitates the reduction of another compound and is itself oxidized in the process. -common oxidizing agents almost all contain oxygen or a similarly electronegative element. -common reducing agents often contain metal ions or hydrides H-

Incomplete octet

Are stable with fewer than eight electrons in their valence shell and include Hydrogen, (stable with 2 electrons), Helium (2), Lithium (2), Beryllium (4), and Boron ( 6)

Compare and contrast the three definitions for acids and bases; Arrhenius Brosted-Lowry Lewis

Arrhenius acid= Dissociates to form excess H+ in solution Arrhenius base= Dissociates to form excess OH- Bronsted-Lowry acid= H+ donor Bronsted-Lowry base= H+ acceptor Lewis acid = Electron pair acceptor Lewis base= Electron pair donor

Arrhenius acids

Arrhenius acids dissociate to produce an excess of H+ hydrogen ions in solution. acids contain H at the beginning of their formula

Arrhenius bases

Arrhenius bases dissociate to produce an excess of OH- hydroxide ions in solution. Bases contain OH at the end of their formula

Although the octet rule dictates much of molecular structure, some atoms can violate the octet rule by being surrounded by more than eight electrons. Which of the following is the best explanation for why some atoms can exceed the octet.?

Atoms that exceed the octet can do so because they have d- orbitals in which extra electrons can reside. All atoms in the third period or greater have d-orbitals which can hold an additional 10 electrons. The typical octet electrons raided in the s- and p-orbital but elements in period 3 or higher can place electrons into these d-orbitals.

Acetic acid dissociates in solution according to the following equation; CHCOOH = CH3COO- + H+ If sodium acetate, is added to a solution of acetic acid in excess water, which of the following effects would be observed in the solution? Decreased pH Increased pH Decreased pKeq (pKa) Increased pKeq (pKa)

B. Increased pH Adding sodium acetate increase the number of acetate ions present. According to Le Chateliers principle this change willl push this reaction to the left, resulting in a decrease in the number of free H+ ions. Because pH is determine by the hydrogen ion concatenation, a decrease in the number of free protons will increase the pH,. An acids Ka will remain constant under a given temperature and pressure, eliminating C & D

Which of the following bases is the weakest? KOH NH3 CH3NH2 Ca(OH)2

B. NH3 Soluble hydroxides of Group IA and IIA metals are strong bases, eliminating choices A & D. B & C are both weak bases; however methyl amine contains an alkyl group, which is electron donating. This increases the electron density on the nitrogen in methyl amine, making it a stronger (Lewis) base. Therefore, ammonia is the weakest base.

The function of a buffer is to; Maintain a neutral pH Resist changes in pH when small amount of acid or base are added Slow down reactions between acids and bases Speed up reactions between acids and bases

B. Resist changes in pH when small amount of acid or base are added The purpose of a buffer is to resist changes in the pH of a reaction. Buffers are not generally used to affect the kinetics of a reaction. so C & D are incorrect. A is correct only in specific circumstances where the pH of the buffer solution itself is neutral. Many natural buffer systems maintain pH in the acidic or basic ranges.

Increasing temperature can alter the Keq of a reaction. Why might increasing temperature indefinitely be unfavorable for changing reaction conditions? The equilibrium constant ahs a definite limit that cannot be surpassed The products or reactants can decompose at high temperatures Increasing temperature would decrease pressure, which may or may not alter reaction conditions. If a reaction is irreversible, its Keq will resist changes in temperature.

B. The products or reactants can decompose at high temperatures At extremely high temperatures, reactants or proudest may decompose, which will affect the equilibrium and potentially destroy the desired products. A implies that reactions have limits. which is true, however this does not make increasing temperature unfavorable. C is false because increasing temperature would also increase pressure, assuming constant volume. D is incorrect because it refers to properties of irreversible reactions which would not be involved in an equilibrium between products and reactants

If a certain metal metal multiple oxidation states, its acidity as an oxide generally increases as the oxidation state increases. Therefore, which of the following tungsten compounds is likely to be the strongest acid? WO2 WO3 W2O3 W2O5

B. WO3 11 on 393

Which of the following equations describe the maximum number that can fill a subshell? 2l+2 4l+2 2l^2 2l^2+2

B.) 4l + 2 this describes the azimuthal quantum number l which ranges from 0 to n-1 with n being the principal quantum number. look at this S - l=0, # of electron = 2 P- l=1, # of electron = 6 D- l=2, # of electron = 10 F- l=3, # of electron = 14 4*l +2 4*3 +2 = 14

Which of the following quantum number sets are possible? -n=2; l=2; ml= 1; ms= +1/2 -n=2; l=1; ml= -1; ms= +1/2 -n=2; l=0; ml= -1; ms= -1/2 -n=2; l=0; ml= 1; ms= -1/2

B.) azimuthal quantum number (-l) cannot be higher than -1 leaving only B & C. ml the magnetic property can only be an integer value between -l and l. It cannot be equal to +1 if l=0 which rules out C & D

Bond Length and how they can differ

Bond length decrease as the bond order increases, they also decrease with larger differences in electronegativity. Bond length decrease as moving to the right along the periodic tables rows because more electronegative atoms have shorter atomic radii Example HCN and C2H2: HCN is more likely to hold its electrons closer than the carbons in C2H2 meaning shorter radius

How does the transition state theory compare with the collision theory of chemical kinetics? Transition State theory Collision theory

Both theories require a certain activation energy to be overcame in order for a reaction to occur (therefore not all reactions will occur) The transition state theory focuses on forming a high energy activated complex that can then proceed forward of backward, foreign the products or reverting to the reactants, respectively. The collision theory focuses on the energy and orientation of reactants, and considers each potential reaction to be "all-or-nothing" (either there is enough energy to form the products or there is not)

Bronsted-Lowry acids

Bronsted-Lowry acids are species that can donate hydrogen ions.

Bronsted-Lowry bases

Bronsted-Lowry bases are species that can accept hydrogen ions.

Buffer solutions are what?

Buffer solutions consist of a mixture of a weak acid and its conjugate salt or a weak base and its conjugate salt; they resist large fluctuations in pH

Compoudn A has a Ka (equilibrium constant of acid dissociation) of approxiamately 10^-4. Which of the following compounds is most likely to react with a solution of compound A? HNO3 NO2 NH3 N2O5

C. NH3 Ka is equal to the ratio of product to reactants, with each species raised to its stoichiometric coefficient. A compound with a Ka greater than 10^-7 contains more H+ cations than HA^- anions at equilibrium, which makes it an acid. This means that the compound in question is likely to react with a compound that tis basic. OF the four answer choice NH3 is the only base.

In the compound KH2PO4, which element has the highest oxidation number? K H P O

C. P #10 on 393

FeI (aq) + I2 (g) = FeI3 (aq) If this reaction were exothermic, what effect would decreasing the temperature have on the equilibrium? The forward reaction rate and the reverse reaction rate both increase. The forward reaction rate decreases while the reverse rate increases The forward reaction rate increases while the reverse reaction rate decreases The forward reaction rate and the reverse reaction rate both decrease

C. The forward reaction rate increases while the reverse reaction rate decreases An exothermic reaction produces heat. Decreasing the temperature factors product formation, resulting in an increase in the forward reactions rate with a concomitant decrease in the reverse reaction rate.

A reaction is found to stop just before all reactants are converted to products. Which of the following could be true about this reaction? The reaction is irreversible and the forward rate is greater than the reverse rate The reaction is irreversible and the reverse rate is too large for products to from. The reaction is reversible, and the forward rate is equal to the reverse rate. The reaction is reversible, and the reverse rate is greater than the forward rate

C. The reaction is reversible, and the forward rate is equal to the reverse rate. This scenario likely describes a situation in which a reaction has reached equilibrium very far to the right (with high product concentration and low reactant concentration). This reaction must be reversible because the reaction did not proceed all the way to the right. Any reaction in equilibrium has equal forward and reverse rates of reaction.

Potentiometry in an oxidation-reduction titration is analogous to performing an acid-base titration with acidic indicator basic indicator pH meter oxidizing agent

C. pH meter 13 393

At what pH is the second equivalence point pH= 3.0 pH= 4.1 pH= 5.9 pH= 7.2

C. pH= 5.9 The second equivalence point is the midpoint of the second steep increase in slope. This corresponds to approximately pH=5.9

Balance the equation C4H10 + 14 O2 --> CO2 + H2O

C4H10 + O2 --> CO2 + H2O Balance the Carbons first (4 on each side) C4H10 + O2 --> 4 CO2 + H2O Balance the Hydrogens (10 on each side) C4H10 + O2 --> 4 CO2 + 5 H2O Balance the Oxygens ( 13 on product side) C4H10 + 13/2 O2 --> 4 CO2 + 5 H2O Multiply by 2 to obtain a whole number ratio 2 C2H10 + 13 O2 --> 8 CO2 + 5 H2O

Rank the following elements by decreasing first ionization energy Ca C Ge K

C>Ge>Ca>K

Which of the following molecules contains the oxygen atom with the most negative formal charge? H2O CO3^2- O3 CH2O

CO3^2- Use resonance and formal charge when comparing. CO3^2- gives 3 resonance structures Each of the three Oxygens carrie a formal charge of -1 in two out of the three structures. avg = -2/3 on each Oxygen Water and Formaldehyde do not have formal charge on the oxygen. O3 or ozone has a -1/2 on two of the three oxygens and a +1 on the central Oxygen

Which of the following best describes the purpose of a catalyst? Catalysts are used up in the reaction increasing reaction efficiency. Catalyst increase the rate of the reaction by lowering the activation energy. Catalysts alter the thermodynamics of the reaction to facilitate the formation of products or reactants Catalysts stabilize the transition state by bringing it to a higher energy.

Catalyst increase the rate of the reaction by lowering the activation energy. This is a direct definition of a catalyst Catalysts are not used up in a reaction, nor do they alter the equilibrium of a reaction. Catalysts also stabilize the transition state by lowering its energy not raising it.

Ions : Cation and Anions

Cation = + (usually metals) Anion = - (usually nonmetals) Ionic compounds are held together by ionic bonds which rely on the forces of electrostatic attraction between oppositely charged particles

5.) The process of formation of a salt solution can be better understood by breaking the process into three steps. 1.Breaking the solute into its individual components 2. Making room for th eoslute in the solvent by overcoming intermolecular forces in the solve 3. Allowing solute-solvent interactions to occur to from the solution Which of the flowing correctly lists the enthalpy changes for these three steps, respectively? Endothermic, exothermic, endothermic Exothermic, endothermic, endothermic Exothermic, exothermic, endothermic Endothermic, endothermic, exothermic

D. Endothermic, endothermic, exothermic The first step will most likely be endothermic because energy is required to break molecules apart. The second step is also endothermic because the intermolecular forces in the solvent must be overcome to allow incorporation of solute particles. The third step will most likely be exothermic because polar water molecules will interact with the dissolved ions, creating a stable solution and releasing energy.

During the assigning of oxidation numbers, which of the following elements would most likely be determined last? Ar F Sr Ir

D. Ir #8 on 393

2.) Which phases of solvent and south can form a solution? I. Solid solvent, gaseous solute II. Solid solvent, solid solute III. Gaseous solvent, gaseous solute

D. Solid solvent, gaseous solute, Solid solvent, solid solute and Gaseous solvent, gaseous solute All three choices can make a solution as long a s the two components create a mixture that is of uniform appearance (homogenous). Hydrogen in platinum is an example of a gas in a solid. Brass and steel are examples of homogenous mixtures of solids. The air we breathe is an example of a homogenous mixture of gases; while these are more coolly simply referred to as mixtures, they still fit the criteria of a solution.

Arrhenius Equation extended

DO NOT MEMORIZE just remember the relationship k = Ae ^ (-Ea/RT) the relationship between A (frequency factor) & k (rate constant) is evident in the equation As the frequency factor of the reaction increases, the rate constant of the reaction also increases in a direct relationship Also if the Temperature (T in kelvin) of a system was to increase to infinity, and all others held constant then the exponent would have a magnitude of less than 1. As the magnitude of the exponent gets smaller it actually is moving from a negative value to zero because it is negative overall. This becomes less negative or more positive. Which means that the rate constant increases. Take away: rate constant increases (positively) as the temperature increases Frequency factor can be increased by increasing the number of molecules in a vessel. More molecules = more collisions

What would happen in this situation: 2C (s) + O2(g) = 2CO (g) the pressure of the reaction vessel is decreased.

Decreasing the pressure of 2C (s) + O2(g) = 2CO (g): reaction shifts right, favoring the side with more moles of gas.

Le Chateliers Principle

Le Chateliers Principle states that when a chemical system experiences a tree, it will react so as to restore equilibrium

Magnetic qauntum number

Denoted by ml For any value of l there will be 2l+1 possible values for ml. For any n, this produces n^2 orbitals. For any value of n there will be a max of 2n^2 (2 electrons per orbitals)

Principal Quantum Number

Denoted by the letter "n", can take on any positive integer value. the larger the integer value of "n" the higher the energy level and radius of the electron' s shell. max amount per shell- 2n^2 the energy between shells decrease as the distance from the nucleus increases N= row that it is in = sub level of electron

Nonpolar covalent bonding

Diatomic atoms such as H2, N2, O2, F2, Cl2, Br2, and I2. Each atom has 7 electrons in its valence shell and by sharing one of these electrons from each atom, they can each form an octet- individually bonded molecule

Which of the following types of intermolecular forces provides the most accurate explanation for why noble gases can liquefy?

Dispersion Forces- Noble gases are entirely uncharged and do not have polar covalent bonds, ionic bonds, or dipole moments . Therefore leave london dispersion forces. These are necessary in condensation

Disproportionation (dismutation) reactions

Disproportionation (dismutation) reactions are a type of redox reactions in which one element is both oxidized and reduced, forming at least two molecules containing the element with different oxidation states.

Double Displacement (metathesis) reactions

Double Displacement (metathesis) reactions income the switching of counterions. These are usually oxidation-reduction reactions because all ions retain their oxidation state.

Lewis acid

Lewis acid is defined as an electron pair acceptor

Explain Hydrogen Bonding

Electronegative atoms bonded to hydrogen disproportionately pull covalently bonded electrons toward themselves, which leaves hydrogen with a partial positive character. Partial Positive charges are attractive to other negative or partial negative

Lewis bases

Lewis bases are electron-pair donors

Electronegativity

Electronegativity values are related to ionization energies: the lower the ionization energy the lower the electronegativity and vice versa same trend as IE

The properties of atoms can be predicted to some extent by their location within the periodic table. Which property or properties increase in the directions of the arrows shown? Arrows going up and right. I. Electronegativity II. Atomic Radius III. First Ionization energy

Electronegativty and First Ionization Energy this makes it more difficult to remove an electron. Atomic radius decreases as you go up and right

Difference between Electronic Geometry and Molecular Geometry

Electronic Geometry- describes the spatial arrangement of all pairs of electrons around the central atom, including both the bonding and the lone pairs. Molecular Geometry- describes the spatial arrangement of only the bonding pairs of electrons

Double Displacement Reactions

Element from two different compounds switch places and form two new compounds CaCl2 + 2 AgNO3 --> Ca(NO3) + 2 AgCl

When an electron drops from Excited state to Ground state what happens?

Emission form this change will yield fluorescence, color from emitted light. When this happened and each are returning to the ground state each will emit a photon with wavelength of the specific energy transition

Empirical vs. Molecular formulas

Empirical is simplest whole ratio formula (CH) Molecular is exact what is in it (C6H6

Chapter 6

Equilibrium

Equivalence point is indicated by..

Equivalence point is indicated by the steepest slope in a titration curve; it is reached when the number of acid equivalents in the original solution equals the number of base equivalents added or vice verse

Equivalent is deined as what?

Equivalent is defined as one mole of the species of interest.

Concept on acids

Every Arrhenius acid (or base) can also be classified as Bronsted-Lowry acid (or base). Every Bronseted-Lowry acid (or base) can also be classified as a Lewis acid (or base). This logic does not always work the other way (for example, NH3 is a Bronsted-Lowry base, but not an Arrhenius base)

imagine a graph has two humps in it starting at A and going up to B as a peak of one hump then going to a trough of C and up to D as another peak and the down to E the lowest point of the graph. The overall reaction depicited by this energy diagram is: Endergonic, because point B is higher than point A Endergonic, because point C is higher than point A Exergonic, because point D is higher than point E Exergonic, because point A is higher than point E

Exergonic, because point A is higher than point E A system is exergonic if energy is released by the reaction. For exergonic reactions, the net energy change is negative and free energy of the final products is lower than the free energy of the initial reactants. Point E which represents the final products is lower on the energy diagram than point A, which represents the energy of initial reactants. Thus energy must have been given off and the reaction is exergonic

When dissolved in water, which of the following ions is most likely to form a complex ion with H2O. Na+ Fe2+ Cl- S2-

Fe2+ Iron is a transition metal, transition metals can often form more than one ion. Iron for example can be Fe2+ or Fe3+ Transition metals in various oxidation states can often form hydration complexes with water. There aren't any other transition metals so this is the pick.

For metel elements that can form more than one positive ion the charge is indicated by a roman numeral

Fe2+ = Iron (II) Fe3+ = Iron (III) Cu+ = Copper (I) Cu2+ = Copper (II)

using -ous and -ic to indicate the charge

Fe2+= Ferrous Fe3+ = Ferric Cu+ Cuprous Cu2+ = Cupric ous- less charge ic- more charge

Find Empirical and molecular formulas of a compound that contains 40.9% C, 4.58% H, and 54.52% O. With molar mass of 264

First convert each to Mol in 100 gram sample 40.9/12 = 3.4 4.58/1 = 4.6 54.52/16 = 3.4 Find ratios using smallest mol found C: 3.4/3.4 = 1 H 4.6/3.4= 1.33 O 3.4/3.4 = 1 convert the numbers into whole numbers so multiply each so each will turn in to a whole C: 1x3 = 3 H: 1.33 x 3 = 4 O: 1 x 3 = 3 C3H4O3 (EMPIRICAL) 3x12 + 4x1 + 3x16 = 88 264 / 88 = 3 C3H403 x 3 = C9H12O9 (MOlECULAR)

To calculate the formal charge

Formal charge is the difference between the number of electrons assigned to an atom in a lewis structure and the number of electrons normally found in the atoms valence shell. Formal Charge= V-N nonbonding - 1/2 N bonding V= normal number of electron N nonbonding= number of nonbonding electrons N bonding = number of bonding electrons remember to double the amount of bond since each bond is two electrons

Example of formal charge and oxidation number CO2

Formal charge on each of the molecules would be 0 Oxidation numbers of each of the oxygen atoms is -2 and of the carbon is +4

Exceptions to the octet rule

Hydrogen Helium Lithium Beryllium and any elements in or beyond the third period may be exceptions because they can take on more than eight electrons in their valence shells Remember for Test day that atoms can have more than four bonds so don't discount any with 5!!

Formation of what in solution greatly increases solubility?

Formation of a complex ion in solution greatly increases solubility. -the formation or stability constant (Kf) is the equilibrium constant for complex formation. Its value is usually much greater than Ksp. -The formation of a complex increases the solubility of other salts containing the same ions because it uses up the products of those dissolution reactions, shifting the equilibrium to the right -The common ion effect decreases the solubility of a compound in a solution that already contains one of the ions in the compound. The presence of that ion in solution shifts the dissolution reaction to the left, decreasing its dissociation.

Formation of complex ions increase what?

Formation of complex ions increases the solubility of otherwise insoluble ions (the opposite of the common ion effect) The process of forming a complex ion involves electron pair donors and electron pari acceptors such as those seen in coordinate covalent bonding.

Spontaneous & nonspontaneous reactions

Gibbs free energy (deltaG) determines whether or not a reaction will occur by itself without assistance. remember that just because a reaction is spontaneous doesn't mean it will run quickly

If you mix 180 g of the following compounds in 250 mL of water (density = 1 g/mL) what are their concentrations in molality, molarity, and normality (for acid-base chem)? Glucose 180 g/mol Carbonate 60 g/mol

Glucose 180 g/mol Molality = 1 mol/ 25 kg solvent = 4m Molarity = cannot be calculated with information given by close to molality Normality= 0N (glucose is a neutral compound) Carbonate 60 g/mol Molality = 3 mol / .25kg solvent = 12m Molarity = cannot be calculated with information given but close to molality Normality = approximately 24N (twice the molality which is close to 12M)

Charges of the element groups

Group 1 and Group 2 = have charges of +1 and +2 Nonmetal which are on right side of the table form anions examples starting with Boron and carbon (group 3 and 4) can have +3. Group 5 ( Nitrogen) has -3 Group 6 (Oxygen) has -2 Group 7 -ines ( florine) has -1 group 8 (Helium) 0 charge

Name two ions that form salts that are always soluble.

Group I metals, ammonium, nitrate, and acetate salts are always soluble

Using ide

H- = Hydride F- = Fluoride O2- = Oxide S2- = Sulfide N3- = Nitride P3- = Phosphide

Identify the conjugate acid-base parking in the reactions below: H2CO3+H2O = HCO3- + H3O- H2PO4-+H2O = H3PO4 + OH-

H2CO3+H2O = HCO3- + H3O- Acid = H3CO3 Base= H20 Conjugate Acid= H3O+ Conjugate Base= HCO3^- H2PO4-+H2O = H3PO4 + OH- Acid = H2O Base= H2PO4^- Conjugate Acid= H3PO4 Conjugate Base= OH-

Conjugate acid-base pairs

H3O+ is the conjugate acid of H2O Oh- is the conjugate base of H2O

Identify which reactants are amphoteric species in the following reactions. For those species, determine if the compound is also amphiprotic. HCO3+ HBr = H2CO3 + Br- 3HCL + Al(OH)3 = AlCl3 + 3H2O 2HBr + ZnO = ZnBr2 + H2O

HCO3- + HBr = H2CO3 + Br- Amphoteric Reactant = HCO3- This is Amphiprotic 3HCL + Al(OH)3 = AlCl3 + 3H2O Amphoteric Reactant = Al(OH)3 This is not Amphiprotic 2HBr + ZnO = ZnBr2 + H2O Ampoteric Reactant = ZnO This is not Ampiprotic

What is the pH of a solution with [HClO4] = 10 M?

HClO4 is a strong acid, it will fully dissociate in solution. [H+] = 10M pH= -log[H+] = -log 10M = -1

Half equivalence point is what?

Half-Equivalence point is the midpoint of the buffering region in which half of the titrant has been protonated (or deprotonated); Thus, [HA] = [A-] and a buffer is formed

Which of the following atoms only has varied electrons in its ground states? Sodium Iron Cobalt Helium

Helium atoms at the end of each block of s, p, d. and f have paired electrons

Henderson-hasselbalch Equation

Henderson-Hasselbalch equation quantifies the relationship between pH and pKa, for weak acids and between pOH and pKb of weak bases; when a solution is optimally buffered, pH= pKa and pOH = pKb

If a compound has a Ka value >> water, what does it mean about its behavior in solution? How does this compare with a solution that has only a slightly higher Ka than water?

High Ka indicates a strong acid, which will dissociate completly in solution. Having a Ka slightly greater than water means the acid is a weak acid with minimal dissociation

If a compound has a Kb value >> water, what does it mean about its behavior in solution? How does this compare with a solution that has only a slightly higher Kb than water?

High Kb indicates a storng base, which will dissociate completly in solution. Having a Kb slightly greater than water means the base is a weak base with minimal dissociation.

What determines the length of an elements atomic radius? I. The number of valence electrons II. The number of electron shells III. The number of neutrons in the nucleus `

I. & II. The number of valence electrons and the number of electron shells Valence electrons for example as one moves across a period valence electron are added and the electron are more strongly attracted to the central protons. This shrinks the radius. As you go down the column more shells are added and the electrostatic forces are weekend and the atomic radius increases.

A certain chemical reaction has the following rate law:: rate = k[NO2][Br2] Which of the following statements necessarily describes the kinetics of this reaction? I. The reaction is second order II. The amount of NO2 consumed is equal to the amount of Br2 consumed III. The rate will not be affected by the addition of a compound other than NO2 and Br2

I. The reaction is second order If the sum of the exponents (orders) of the concentrations of each species in the rate law is equal to 2, then the reaction is second-order. The exponents in the rate law are unrelated to stoichiometric coefficients, so NO2 and Br2 could have any stoichiometric coefficientsin the original reaction and still be a second-order reaction, invalidating II. & III>

Ionization Energy

IE or ionization potential is the energy required to remove an electron from a gaseous species. This requires an input of heat which makes it an endothermic process. The greater the IE, the closer the valence electrons are to the nucleus, and the more tightly bound they are. which increases from left to right and bottom to top

What is the normality of a 2 M Mg(OH)2 solution?

Identify hydroxide ions (OH-) - 2 Normality= molarity x n(OH)= 2 M x 2 (mol OH-/mol Mg(OH)2)

Combination reactions

In combination reactions, two or more species come together to form a product.

Combustion reactions

In combustion reactions, a fule (usually hydrocarbon) is mixed with an oxidant (usually oxygen), forming carbon dioxide and water.

Decomposition reactions

In decomposition reactions, one product breaks down into two or more species.

In the Bronsted-Lowry deffiniton acids have what?

In the Bronsted-Lowry deffiniton acids have conjugate bases that are formed when the acid is deprotonated. Bases have conjugate acids that are formed when the base is protonated. Strong acids and bases have very weak(inert) conjugates Weak acids and basses have weak conjugates

A reactant in a second order reaction at a certain temperature is increased by a factor of 4. By how much is the rate of the reaction altered? It is unchanged It is increased by a factor of 4 It is increased by a factor of 16 It cannot be determined from the information given

It cannot be determined from the information given A second order reaction can be second order with respect to one reactant or first order with respect to two different reactants. In this case one reactant was increased by a factor of 4. If the reaction is second order with respect to this reactant, the rate will increase by a factor f 16. If it is first order with respect to this reactant the rate will increase by a factor of 4

The concentrations of all reactants in a zero order reaction are increased two fold. What is the new rate of the reaction? It is unchanged It is decreased by a factor of 2 It is increased by a factor of 2 It cannot be determined from the information given

It is unchanged by definition zero-order reactions are unaffected by the concentrations of any reactants in the reaction. Thus changing the concentrations of these reactants will not affect the rate

In a third order reaction involving two reactants and two products, doubling the concentration of the first reactant causes the rate to increase by a factor of 2. What will happen to the rate of this reaction if the concentration of the second reactant is cut in half? It will increase by a factor of 2 It will increase by a factor of 4 It will decrease by a factor of 2 It will decrease by a factor of 4

It will decrease by a factor of 4 the rate is first order with respect to the concentration of the first reactant; when the concentration of that reactant doubt the rate also doubles. The reaction order with respect to the other reactant much be 3-1= 2 if the concentration of this second reactant is multiplied by 1/2 the rate will be multiplied by (1/2)^2= 1/4

What is the mathematical relationship between Ka, Kb and Kw?

Ka x Kb= Kw

On a reaction coordinate diagram, how would the kinetic pathway appear as compared to the thermodynamic pathway?

Kinetic pathway require a smaller gain in free energy to reach the transition state. They also have a higher free energy of the products, with a smaller difference in free energy between the transition state and the products.

What conditions favor formation of a kinetic product? A thermodynamic product?

Kinetic products are favored at low temperatures with low heat transfer. Thermodynamic products are favored at high temperatures with high heat transfer.

Kinetic products are what?

Kinetic products are higher in free energy than thermodynamic products and can form at lower temp. These are sometimes termed "fast" products because they can form more quickly under such conditions.

Which of the following correctly ranks the compounds below by ascending boiling point? Acetone KCl Kr Isopropyl Alcohol

Kr < Acetone < Isopropyl Alcohol < KCl Kr is a noble gas with a full octet so the only forces present are London dispersion forces Acetone and Isopropyl Alcohol are both polar so both have dipole- dipole interactions which are stronger than dispersion forces Isopropyl Alcohol can also hydrogen bond increasing the Hydrogen Bonding the strongest which are ionic bond exist in potassium chloride

What is Kw dependent on?

Kw is only dependent on temperature Changes in concentration, pressure or volume will not affect Kw Kw changed when temp does will change the significance of the pH scale.

LEO the lion says GER

LEO the lion says GER Loss of Electrons is Oxidation, Gain of Electrons is Reduction

LEORA says GEROA

LEORA says GEROA Loss of Electrons is Oxidation; (Reducing Agent), Gain of Electrons is Reduction (Oxidizing Agent)

potential energy diagram

Remember if you are given a diagram with different levels of reactions but with identical conditions and asked to state which one finishes first... The first one finished is reaction with the lowest activation energy so will have the fastest rate. That is if the ending line is above all of the other and above is starting point. the slowest one will have the line farthest away from the starting point

Which of the following experimental methods should NEVER affect the rate of a reaction? Placing an exothermic reaction in an ice bath Increasing the pressure of a reactant in a closed container Putting the reactants into an aqueous solution Removing the product of an irreversible reaction

Removing the product of an irreversible reaction Temperature does affect rate constant k changing the pressure of a gas will affect the number of effective collision per time Solvents do affect the rate of reactions depending on how the reactants interact with the solvents But, removing it from a irreversible reaction should not affect the rate of the reaction because the rate law does not depend on the concentration of products

What would happen in this situation: H3PO4 (aq) + H2O (l) = H3O^+ (aq) + H2PO4^- (aq) water is removed (without changing temp)

Removing water from H3PO4 (aq) + H2O (l) = H3O^+ (aq) + H2PO4^- (aq): Reaction shifts left.' All concentrations would increase proportionally ; because there re more products that reactants the value of Q will increase.

Reversible reaction

Reversible reactions eventually reach a state in which energy is minimized and entropy is maximized -chemical equilibria are dynamic- the reactions are still occurring, just at a constant rate. -the concentrations of reactants and products remain constant because the rate of the forward reaction equals the rate of the reverse reaction.

Saturated solution are in what at a particular temperature?

Saturated solutions are in equilibrium at that particular temperature.

Describe the difference between solubility and saturation

Solubility is the among of solute contained in a solvent. Saturation refers to the maximum solubility of a compound at a given temperature; one cannot dissolve any more of the solute just by adding more at this temperature.

Solubility is what?

Solubility is the maximum amount of a solute that can be dissolved in a given solvent at a given temperature; it is often expressed as molar solubility - the molarity of the solute at saturation

What is one way in which solubility of a compound can be increased?

Solubility of solids can be increased by increasing temperature. Solubility of gases can be increased by decreasing temperature or increasing the partial pressure of the gas above the solvent (henry's law)

Chapter 9

Solutions

Solutions are what?

Solutions are homogenous mixtures composed of two or more substances. -they combine to form a single phase, generally the liquid phase.

Solvation in water is called what?

Solvation in water can also be called hydration,

Describe the process of solvation.

Solvation referes to the breaking of intermolecular forces between solute particle and between solvent particles, with formation of intermolecular forces between solute and solvent particles. In an aqueous solution, water is the solvent.

Solvent particels do what?

Solvent partivels surround solute particels via electrostatic interactions in a process called solvation or dissolution

Boiling point depression

The boiling point elevation formula calculates the among that the normal boiling point is raised. The value calculated is not he boiling point itself. (delta) Tb= iKbm (delta) Tb = is the increase in boiling point i= vant hoff factor = number of particles into which a compound dissociates in solution. NaCl = ( i=2 ) Covalent molecules such as glucose have i values of 1 Kb= proportionality constat characteristic of a particular solvent (is provided) m = is the molality of the solution

Which atom has the most positive charge in [PO4]^3-?

The phosphorus atom does. The phosphate molecule has an overall formal charge of -3. The four oxygen atoms would each be assigned a formal charge of -1. Given the overall charge of -3 and the -1 charge on each oxygen, the phosphorus must have a formal charge of +1

imagine a graph has two humps in it starting at A and going up to B as a peak of one hump then going to a trough of C and up to D as another peak and the down to E the lowest point of the graph. Which process has the highest activation energy? The first step of the forward reaction The first step of the reverse reaction The second step of the forward reaction The second step of the reverse reaction

The first step of the reverse reaction the activation energy of a reaction is represented by the distant on the y-axis from the energy of the reactant to the peak energy prior to formation of products. The activation energy of the first step of the forward reaction, for example is equal to the distance along the y-axis from point A to point B. The largest energy increase on this graph occurs during the progress from point E to point D, which represents the first step of the reverse reaction.

Reaction concentrations affecting reaction rates

The greater the concentrations of the reactants, the greater the number of effective collisions per unit times. Recall the Arrhenius equation, increase in concentration leads to increase in frequency factor (A), therefore the reaction rate will increase for all but zero-order reactions for gas reactions, the gas reactants serve as a measure of concentration

The law of mass action-

The law of mass action gives the expression for the equilibrium constant, Keq. The reaction quotient Q has the same form but can be calculated at any concentrations of reactants and products. -Q is a calculated value that reflects the reactant and product concentrations at any given time during a reaction. -Keq is the ratio of products to reactants at equilibrium, with each species raised to its stoichiometric coefficient. Keq for a reaction is constant at a constant temp. Pure solid and liquids do not appear in the law of mass action; only gases and aqueous species do.

Which is an important property of bond energy?

The more shared electron pairs comprising a bond, the higher the energy of that bond. Hence a triple bond is short meaning high energy and order. Double bond is medium and single is lowest energy.

What would increasing the concentration of reactants accomplish in a solution containing a saturated catalyst? It would increase the rate constant but not the reaction rate It would decrease the rate constant but increase the reaction rate It would increase the rate constant and increase the reaction rate The reaction rate would be unaffected

The reaction rate would be unaffected While increasing the concentration of reactants can alter the reaction rate in first- or higher-order reactions, saturated solution containing a catalyst have a maximum turnover tutor and cannot increase the rate constant or the reaction rate any higher by adding more reactant molecules.

What is solubility product constant (Ksp)

The solubility product constant (Ksp) is simply the equilibrium constant for a dissociation reaction

Transition State extended

The transition state, also called the activated complex, has greater energy than both the reactants and the products and is denoted by a symbol (++) Energy required to reach the transition state is the activation energy

Yeilds...

Theoretical yield is the maximum amount of product that can be generated as predicted Actual yield is amount of product one actually obtains during the reaction Ratio of Actual yield to theoretical yield multiplied by 100 percent gives the percent yield.

Thermodynamic products are what?

Thermodynamic products are lower in free energy that kinetic products and are therefore more stable. Despite proceeding more slowly than the kinetic pathway, the thermodynamic pathway is more spontaneous ( more negative (delta) G )

Catalysts

These increase the reaction rate without themselves being consumed in the reaction. Catalyses interact with the reactants either by absorption or through the formation of intermediates and stabilize them so as to reduce the activation energy necessary of the reaction to proceed. They may increase the frequency of collisions between the reactants; change the relative orientation of the reactants, donate electron density to the reactants; or reduce intramolecular bonding within reactant molecules Catalysts decrease the activation energy for both forward and reverse reactions, this means the catalysts only change the rate of the reactions, having no impact on the equilibrium position or the measurement of Keq CAN NOT transform non spontaneous reactions into spontaneous ones only make spontaneous reactions more more quickly toward equilibrium

Transition State Theory

This states that when molecules collide with energy equal to or greater than the activation energy, they form a transition state in which the old bonds are weekend and the new bonds begin to form, then the transition states then dissociates into products forming new bonds. reaction A2 + 2B = 2Ab

Titration is a what?

Titration is a procedure used to determine the concentration of a known reactant in a solution. Ke Titrant has a known concentration and is added slowly to the titrand to reach the equivalence point Titrand has an unknown concentration but a known volume

PCl5

Trigonal Bipyramidal

BH3

Trigonal planar

Orbital diagrams for Nitrogen and Iron (Hund's Rule)

Use the arrows when it is in middle of the row then it isn't forced to double up like seats on a bus.

Vapor pressure depression

Vapor pressure depression follows Raoults law -the presence of other solutes decreases the evaporation rate of a solvent without affecting its condensation rate, thus decreasing its vapor pressure. -Vapor pressure depression also explains boiling point elevation - as the vapor pressure decreases, the temp (energy) required to lil the liquid must be raised.

Key concept

Vapor pressure depression goes hand in hand with boiling point elevation. The lowering of a solution's vapor presser would mean that a higher temp is required to match atmospheric pressure, thereby raising the boiling point.

What would happen in this situation: CH4(g) + 2 O2 (g) = CO2 (g) + 2 H2O (l) + heat the reaction vessel is warmed

Warming CH4(g) + 2 O2 (g) = CO2 (g) + 2 H2O (l) + heat: Reaction shifts left, using the additional heat energy to produce more reactant.

do not do the Practice for NINE

Waste of time

Water dissociation constant Kw is what?

Water dissociation constant Kw is 10^-14 at 298K. Kw is only affected by temperatures

What is and example of Amphoteric, ampiprotic species?

Water is an example of an amphoteric, amphiprotic species, it can accept a hydrogen ion to become a hydronium ion, or it can donate a hydrogen ion to become a hydroxide ion.

What are good example of Amphoteric and amphiprotic substances?

Water, amino acids, and partially deprotonated polyprotic acids such as bicarbonate and bisulfate are common example of amphoteric and amphiprotic

When balancing redox reactions

When balancing redox reactions, the half-reaction method, also called the ion-electron method, is the most common. -Separate the two half-reactions -balance the atoms of each half-reaction, Start with all the elements besides H and O. In acidic solution, balance H and O using water and H+. In basic solution, balance H and O using water and OH- -Balance the charges of each half-reaction by adding electrons as necessary to one side of the reaction. -Multiply the half-reactions as necessary to obtain the same number of electrons in both half-reactions. -Add the half-reactions, canceling out terms on both sides of the reaction -Confirm that the mass and charge are balanced.

Weak acids and bases do what

Weak acids and bases do not completly dissociate in solution and have coresponding dissociation constants Ka & Kb

Rank the following by increasing atomic radius Nd Pr Ta Xe

Xe<Nb<Ta<Pr

Reaction orders

Zero-order First-order Second-order higher-order mixed-order

In a certain equilibrium process, the activation energy of the forward reaction is greater than the activation energy of the reverse reaction. This reaction is .... endothermic exothermic spontaneous nonspontaneous

endergonic/nonspontaneous first draw the energy diagram have Ea of forward being bigger that Ea of reverse if the activation of the forward reaction is greater than the activation energy of the reverse reaction then the products must have a higher free energy than the reactants. The overall energy of the system is higher at the end than it was in the beginning. The net free energy change is positive indicating an endergonic (non spontaneous) reaction Endothermic and exothermic are associated with enthalpy.

Atomic Mass

equal to the mass of the proton or neutron of the atom Amu is equal to the atomic mass which is under the element on the p table atoms of the same element with varying mass numbers are called isotopes

Percent Composition of Cr in K2Cr2O7

first find molar mass 2 x 39.1 + 2 x 52 + 7 x 16 =294 Percent comp of Cr in K2Cr2O7 2x52 / 294 = .354 x 100 = 35.4

gram equivalent weight of sulfuric acid

first find molar mass of H2SO4 2 x 1 (g/mol) + 1 x 32.1 (g/mol) + 4 x 16 (g/mol) + = 98.1 (g/mol) Protons in H2SO4 is (2 H) Gram equivalent weight= Molar mass/n 98.1/2(H)= 49.05 (g/mol H+)

Temperature affecting reaction rates

for nearly all reactions, the reaction rate will increase as the temperature increases. temperature of a substance is a measure of the particles average kinetic, increasing the temperature increases the average kinetic energy of the molecules All reaction even the nuclear reaction are temperature dependent and experience an optimal temperature for activity Remember for biological systems, that if you raise the temp by 10 degrees celsius it will result in an approximate doubling of the reaction rate, but is not the case for all reactions.

Sulfate SO4^2-

giving sulfur 12 valence electrons makes it neutral versus giving it 4 single bond to the oxygens which yield +2

First-Order Reaction aA + bB = cC + dD

has a rate that is directly proportional to only one reactant, such that doubling the concentration of that reactant results in a doubling of the rate of formation of the product rate= k[A]^1 or rate = k[B]^1 k has units of s^-1 plotting first order reaction give a curved nonlinear graph plotting ln[A] vs time reveals a straight line the slope of such a line is the opposite of the rate constant k (-slope)

Second-Order reaction

has a rate that is proportional to with the concentrations of two reactants or to the square of the concentration of a single reactant. rate= k[A]^1[B]^1 or rate = k[A]^2 or rate = k[B]^2 k has units of (m^-1 s^-1) second order rate law often suggest a physical collision between two reactant molecules especially if the rate law is first order with respect to each of the two reactants plotting a second order reaction would give the same as first order but plotting 1/A vs time reveals a positive linear graph k=slope

Neutralization Reactions

specific type of double displacement reactions in which an acid reacts with abase to produce a salt and water HCl + NaOH --> NaCl + H2O

Halogens (group 7)

highly reactive nonmetals with seven valence electrons -desperate to complete the octet by gaining one additional electron -Range grom gaseous (F2), to Liquid (Cl2) to solid (I2) -highly reactive to alkali and aline earth metals -Fluorine has highest electronegativity -Usually found as halides or diatomic molecules and not in their elemental state oxidation state -1

Definition of Rate example: 2A + B = C

in which one mole of C can be produced from two moles of A and one mole of B, Rate can be can be described as the disappearance of reactants over time or the appearance of products over time reactants are being consumer so place a negative sign in from of the rate expression for the reactants rate of reaction; with respect to A is -(delta A/delta t) with respect to B is -(delta B/delta t) with respect to C is +(delat C/ delta t) rate of change of concentrations Two moles of A are consumed for every mole of B rate= -delta[A]= 2 x rate -deltat[B] Two moles of A are consumed, only one mole of C is produced rate -delta[A] = 2 x rate deltat [c]

Noble Gases (group 8)

inert gases and have minimal chemical reactivity due to their filled valence shells - high ionization energies - little to no tendency to gain or lose electrons -low boiling points and exist at room temp -can be used for lighting

imagine a graph has two humps in it starting at A and going up to B as a peak of one hump then going to a trough of C and up to D as another peak and the down to E the lowest point of the graph. Point C in this reaction refers to the: reactants products transition state intermediates

intermediates Point C is a valley and all intermediates exits as valleys reactants are represented by point A (beginning) Products are represented by point E (end) Transition states are represented by B & D (peaks)

Combustion Reaction

involves oxidation using O2 or similar fuel CH4 + 2 O2 --> CO2 + 2H2O

Skin 12

kn

Complete the blank cells in the table by utilizing the mathematical relationships between pH, pOH and ion concentrations.

pH of 4.00 = [H30+] of 10^-4 M= pOH of 10 = [OH-] of 10^-10 and is an acid pH of 3.05 = [H30+] of 8.89 x 10^-4 M = pOH of 10.95 = [OH-] of 1.12 x 10 ^-11 and is an acid pH of 8.81= [H30+] of 1.55 x 10^-9 M = pOH of 5.19 = [OH-] of 6.46 x 10^-6 and is a base pH of 8.27 = [H30+] of 5.31 x 10^-9 M = pOH of 5.73 = [OH-] of 1.88 x 10^-6 and is a Base Look at the differences Acid have low pH, lower negative integers in "M" [H30+], high pOH and high integers in their [OH-] Bases have high pH, high negative integers in "M" [H30+], low pOH and low integers in the [OH-]

Just look at 4 on 195

please

Look at titration curve on 364

please

Mole

qauntity of any substance equal to number of particles that are found in 12 grams of carbon-12 = to 6.022 x 10 ^23 or avogadros number

Determination of Rate Law aA + bB = cC + dD

rate is proportional to [A]^x [B]^y rate law is k =[A]^x [B]^y x&y mean the order of reaction k is the reaction rate coefficient or rate constant and the exponents x and y are the orders of the reaction

The following system obeys second- order kinetics 2NO2--> NO3 + NO (Slow) NO3 +CO --> NO2 + CO2 (fast) What is the rate law for this reaction? rate= k[NO2][CO] rate= k [NO2]^2[CO] rate= k {NO2][NO3 rate= k [NO2]^2

rate= k [NO2]^2 Remember that ht slow step of a reaction is the rate determining step!! The rate is always relates to the concentrations of the reactants in the rate determining step (not the overall reaction) so NO2 is the only compound that should be included in the correct answer, The concentration of NO2 is squared in the rate law because the stoichiometric coefficient of NO2 in the rate determining step is 2

The following date is given for the reaction XH4 + 2 O2 --> XO2 + 2 H2O Trial ....[XH4}.....[O2] ..........Rate (M/min) 1...........(.6).........(.6)..............12.4 2..........(.6)..........(2.4)............49.9 3..........(1.2)........(2.4)............198.3 What is the rate law for the reaction described here? rate= k[XH4][O2] rate= k[XH4][O2]^2 rate= k[XH4]^2[O2] rate= k[XH4]^2[O2]^2

rate= k[XH4]^2[O2] In the first two trials the concentration of XH4 is held constant while the concentration of O2 is multiplied by 4. Because the rate of the reaction is also increased by a factor of 4, oxygen must be a first order reactant. In the last two trials, the concentration of O2 is held constant while the concentration of XH4 is doubled. Because the rate of the reaction is increased by a factor of approximately 4 again, XH4 must be a second order reactant. based on this we can conclude the rate law is rate= k[XH4]^2[O2]

Orbitals

regions around the nucleus where electrons are

Rate-determining step

slowest step in any proposed mechanism, acts like a kinetic bottleneck, preventing the overall reaction from proceeding any faster than the slowest step. rate of the entire reaction is only as fast as the rate-determine step

Calculate the Ksp of Ni(OH)2 in water, given that its molar solubility is 5.2 x 10^-6M

these are impossible....

Free energy change of the reaction

this is the difference between the free energy of the products and the free energy of the reactants. negative free energy indicated an exergonic reaction (energy given off) positive free energy change indicates an endergonic reaction (energy is absorbed) transition state exists at the peak of the energy diagram

Collision Theory of Chemical Kinetics

this theory states that molecules must collide with each other to react. This states that the rate of a reaction is proportional to the number of collision per second between the reacting molecules. minimum amount of energy of collision necessary for a reaction to take place is called the activation energy (Ea) Rate= Z x f Z= total number of collisions occurring per second and f is the fraction of collision that are effective or use the Arrhenius equation- k = Ae ^ (-Ea/RT) k= rate constant, A is the frequency factor, Ea is the activation energy, R is the ideal gas constant and T is the temp in Kelvin

Lewis structure

used to show how different atoms can be combined and not useful in showing the geometry of it the amount of dots it has comes from the assigned group numbers.

London Dispersions Forces

weakest of all interactions result of induced dipoles that change and shift moment to moment Strength depends on how easily the electrons are shifted around Larger molecules are easily polarized than the smaller and possess greater dispersion forces

Predict the geometry of CO2

O=C=O = 180 degrees

Resonance structure of SO2

O=S=O O=S-O S would be +1 and O would be =1 O-S=O O would be -1 and S would be +1

Protons

(+) in charge have a mass of approximately one atomic mass unit. atomic number (Z) of an element is equal to the number of protons found in an atom of that element

Electrons

(-) in charge Electrons closer to nucleus are at lower level and further out the higher in energy which are valence electrons. Valence electrons are more likely to react in bonds with other atoms because they experience least electrostatic pull from their own nucleus. These determine reactivity

Neutrons

(=) in charge mass number is indicated by sum of Neutrons and Protons

Using VSEPR on how to determine geometric arrangement of atoms in a compound

- Draw the lewis dot structure -Count the total number of bonding and nonbonding electron paris in the valence shell of the central atom - arrange accordingly

Remember: as electrons go from a lower energy level to a higher energy level, they get AHED

A- Absorb light H- Higher potential E- Excited D- Distant from the nucleus

Suppose an electron falls from n=4 to its ground state, n=1. what is most likely to happen?

A photon is emitted giving off light or fluorescence if it went from ground state to an excited state then a photon is absorbed

SF6

Octahedral

Rank the following elements by increasing electron affinity Ba Cu S Y

Ba<Y<Cu<S

What is the maximum number of electrons allowed in a single atomic energy level in terms of the principal quantum number "n"? 2n 2n+2 2n^2 2n^2+2

C.) 2n^2 2 per orbital. There are only two elements (H & He) that have valence electrons n the n=1 shell. Eight elements (Li to Ne) have electrons in the n-2 shell. this is the only equation that matches this pattern.

CCl4 as an example of a nonpolar compound with four polar bonds

CCl4 has four polar C-Cl bonds but because the molecular geometry of carbon tetrachloride is tetrahedral, the four bond dipoles point to the vertices therefore cancel each other out resulting Symmetrical meaning is is non polar

How electrical geometry and molecular shapes differ

CH4, NH3 and H2O all have a tetrahedral electrical geometry, but differ in molecular shapes such as CH4 is tetrahedral, NH3 is Trigonal Pyriamidal, and H2O is bent

Which particles determine -Charge -Atomic number -Isotope

Charge- Electrons because they can be lost or gained Atomic number- Protons because they never change and are unique to the atom holding the number of protons Isotope- Neutrons because they can have variability between the different atoms

Which of the following species is represented by the electron configuration 1s^2, 2s^2, 2p^6, 3s^2, 3p^6, 4s^1, 3d^5 .

Cr

Planck Quantum Theory

Energy of quantum= E= hv H is proportional constant equal to 6.626x10^-34 v= frequency of the light

Example question: Rydberg Equation The valence electron in a lithium atom jumps from energy level. N=2 to N=4. What is the energy of this transition in Joules? In eV? Note: Rh = 1.097x10^7 m^-1

E= Rh[1/ni^2 - 1/nf^2] = -2.18 x10^-18 [1/2^2 - 1/4^2] = -2.18x10^-18 [3/16] = 4.09 x 10^-19 = -2.55 eV: negative sign means absorption

How to determine the highest electron affinity with the lewis dot structure.

Electron affinity is related to several factors, including atomic size and filling for of the valence shell. As atomic radius increases, the distance between the nucleus and the outermost electrons increases, thereby decreasing the attractive forces between protons and electrons. As a result increased atomic radius will lead to lower electron affinity. If the example has the least amount of electrons needed to be filled it has the highest electron affinity.

If a problem shows arrows in orbital of SPDF going up and down and the last standing straight up which law is being shown?

Hunds rule explains that the electrons fill empty orbital first before doubling up electrons in the same orbital

Which of the following atoms or ions has the largest effective nuclear charge? Cl Cl- K K+

K+ The effective nuclear charge refers to the strength with which the protons in the nucleus can pull on electrons. this helps to explain electron affinity, electronegativity, and ionization energy.

Metal are often used for making wires that conduct electricity. Which of the following properties of metals explains why? Metals are malleable Meals have low electronegativties Metals have valence electrons that can move freely Metals have high melting points

Metals have valence electrons that can move freely All four descriptions are true but the most significant property that contributes to the ability of metals to conduct electricity is the fact that they have valence electrons that can move freely. Malleable just means to shape and does not indicate electricity conduciveness Electronegativity and high melting points do not play a role in conduction of electricity

Predict what the molecular geometry of ammonia,NH3 is

Molecular structure- trigonal pyramidal Electronic structure- Tetrahedral -Central atom, N has three bonding electron paris and one nonbonding electron par for a total of four electron pairs. -four electron pairs will be farthest apart when they occupy the corners of a tetrahedron, because one of the four electron paris is a lone pair, the observed molecular geometry is trigonal pyramidal

H2O as an example of a polar molecule

O-H bond are polar in H2O, each hydrogen atom assuming a partial positive charge and the oxygen assuming a partial negative charge. Asymmetrical meaning it is polar

Example question: Element Q consists of three different isotopes: A, B, and C. Isotope A has an atomic maass of 40 amu and accounts for 60% of naturally occurring Q. isotope B has an atomic mass of 44 amu and accounts for 25 % of Q. Finally, Isotope C has an atomic mass of 41 amu and accounts for 15 % of Q. What is the atomic weight of element Q?

The atomic weight is the weight average of the naturally occurring isotopes of that element. .60 (40amu) + .25 (44amu) + .15 (41amu) = 24amu + 11amu + 6.15 amu = 41.15 amu

Why do halogens often form ionic bonds with alkaline earth metals?

The halogens have much higher electron affinities than the alkaline earth metals. Ionic bonds are formed through unequal sharing of electrons. These bond typically occur because the electron affinities of the two bonded atoms differ greatly. Halogens have high electron affinities because adding a single electron to their valence would create full valence shells. Alkaline earth metals have very low electron affinities and are more likely to be electron donors because the loss of two electrons would leave then with full valence shells.

Dipole moment

The partial positive and negative the separation of positive and negative is called a polar molecule

Atomic Weight

all elements occur as two or more isotopes usually the average of the isotopes

Covalent bonding

an electron pair is shared between two atoms, typically nonmetals, that have relatively similar values of electronegativity. Rather than transferring electrons to form octets, the atoms share electrons. note: if the electron pair is shared equally, the bond is non polar. and if the pair is shared unequally the bond is polar.

Expanded octet

any element in period 3 or greater can hold more than eight electrons including Phosphorus (10), Sulfur (12), Chlorine (14), and many others.

Atomic Radii

atomic radius increases right to left in a period and increases going down a column. The largest radii will be at the bottom of group 1 and belongs to Cesium (Cs)

Polar Covalent Bond

atoms that differ moderately in their electronegativities will share electrons unevenly, resulting in a polar covalent bond

What is the highest-energy orbital of the elements with valence electrons in the n = 3 Shell. s-orbital p-orbital d-orbital f-orbital

d- orbital When n=3, l=0,1, or 2. The highest value for l in this case is 2, which corresponds to the d subshell. n = 3 so the energy shell increase in energy as follows; s < p < d < f

Valence electrons

electrons in the most outer shell, most active and easiest to remove which are available for bonding. Groups 1,2 and 13-18 the valence electrons are found in s- and or p- orbitals Transition elements valence electrons are found in s- and either d- or f- orbital

Spin Quantum Number

fourth quantum number and is denoted by ms electron has two spin orientations designated by +1/2 and -1/2, if in the same orbital they have different spin known as paired. If in different orbital but same ms value then they are known as parallel spins. -1/2 = if arrow is pointing down in pic +1/2 = if arrow is pointing up in picture Tunsten = W = 4 electrons in orbit = +1/2 Gold = Au = 9 electrons in orbit = -1/2 Sulfur = S = 4 electrons in orbit = -1/2 If pairing can happen = (-) No pairing = (+)

Isotopes

have same atomic number but different mass number (varied number of neutron) examples Protium (Hydrogen), atomic number 1 and mass number 1 Deuterium, atomic number 1 and mass number 2 Tritium, atomic number 1 and mass number 3

Ionic Compound characteristics

have very high melting points and boiling points dissolve readily in water and other polar convent and are good conductors of electricityC

quantum numbers

l=0 --> s l=1 --> p l=2 --> d l=3 --> f

Metals

left side and in the middle of the periodic table, this include active metals, transition metals, and the lanthanide and actinide series of elements. lustrous or shiny solids, except for mercury high melting point and densities malleable and ductile metals usually have low effective nuclear charge, low electronegativity, high electropositivity, large atomic radius, small ionic radius, low ionization energy and low electron affinity. metals give up electrons easily

polarity of molecules

lets say two atoms of different electronegativities bond covalently, sharing one or more pairs of electron, the resulting bond is polar, with the more electronegative atom possessing the greater share of the electron density A compound of nonpolar bonds is always non polar A compound with polar bonds may be polar or nonpolar, depending on the spatial orientation o the polar bonds in the molecule If the arrows are pointed towards the central atom from it being partial negative then it is polar

Example: Calculate the charge on the central atom of [NH4]+

lewis structure to the left Nitrogen is in group 5 = 5 valence electrons In NH4 the N has 4 bonds, 8 bonding electrons, 0 nonbonding V=5; Nbonding= 8; Nnonbonding= 0 valence - nonbonding electrons - half of the bonding electrons Formal charge= 5-0-1/2(8) = +1

Which of the following electronic transition would result in the greatest gain in energy for a single hydrogen electron? n=6 to n=2 n=2 to n=6 n=3 to n=4 n=4 to n=3

n=2 to n=6 for it to gain energy it must absorb energy from photons to jump up to a higher energy level. n=2 to n=6 has a bigger jump than n=3 to n=4 n=6 to n=2 loses energy

Which of the following is an important property of the group of elements in group 2 of the periodic table? These element are best electrical conductors in the aperiodic table These elements form divalent cations The second ionization energy for these element is lower than the first ionization The atomic radii of these elements decrease as on moves down the column

represents the alkaline earth metals which form cations or ions with a 2+ charge. All elements in group 2 have 2 electrons in outer most s subshell. alkali metals are the best conductors btw and speaking of ionization the second is always higher than the first and third is higher than the second also radii increases as you go down

Metalloids

semimetals, share some characteristics with both metals and nonmetals the staircase going 3a down the stairs

Alkaline Earth Metals (group 2) basics

share most of the same characteristics with Alkali Metals -slightly smaller atomic radii -2 electrons in their valence shell oxidation state +2

Hunds rule

states that subtle with multiple orbitals (p, d, and f) will fill electrons so that every orbital in a subshell gets one electron before any of them gets a second one

nonmetals

upper right side of the periodic table generally brittle and little to no metallic luster high ionization energies, electron affinities and electronegativities small atomic radii and large ionic radii poor conductors of heat and electricity

Electron configurations

use spectroscopic notation first number denotes the principal energy level, letter designates the subshell superscript gives the number of electrons in that subshell example 2p^4 shows there are 4 electrons in the second p subshell of the second principal energy level.

Ionic bonding

when an element from an atom with low ionzation energy (metal) are transferred to an atom with high electron affinity (nonmetal). forms between atoms that have significantly different electronegativity. Electrons are not shared on an ionic bond NaCl and has a crystalline lattice structure instead of individual bonds. Look at a salt crystal easy to remember that usually ionic bonds occur between group 1-2 with halogen in group 7 halogens gain an electron, metals loosely hold onto it

Example:

within the second principal energy level, n=2. the possible value for l are 0 and 1 also remember that the n-values also tells you the number of possible subshells. therefore, there is only one subshell (l=0) in the first principal energy level: there are two subshells (l=0 and 1) within the second principal energy level: There are three subshells (l=0, 1 and 2) within the third principal energy level and so on.


Ensembles d'études connexes

Chapter 9 - Financial Statements for a Sole Proprietorship

View Set

303 Hinkle PrepU Chapter 39: Assessment of Musculoskeletal Function

View Set

Chapter 7: Founding a Nation, 1783—1791

View Set